What is Conditions: Definition and 1000 Discussions

In mathematical optimization, the Karush–Kuhn–Tucker (KKT) conditions, also known as the Kuhn–Tucker conditions, are first derivative tests (sometimes called first-order necessary conditions) for a solution in nonlinear programming to be optimal, provided that some regularity conditions are satisfied.
Allowing inequality constraints, the KKT approach to nonlinear programming generalizes the method of Lagrange multipliers, which allows only equality constraints. Similar to the Lagrange approach, the constrained maximization (minimization) problem is rewritten as a Lagrange function whose optimal point is a saddle point, i.e. a global maximum (minimum) over the domain of the choice variables and a global minimum (maximum) over the multipliers, which is why the Karush–Kuhn–Tucker theorem is sometimes referred to as the saddle-point theorem.The KKT conditions were originally named after Harold W. Kuhn and Albert W. Tucker, who first published the conditions in 1951. Later scholars discovered that the necessary conditions for this problem had been stated by William Karush in his master's thesis in 1939.

View More On Wikipedia.org
  1. J

    What is going on here? (Heat equation w/ Neumann conditions)

    Homework Statement Solve the heat equation ut=uxx on the interval 0 < x < 1 with no-flux boundary conditions. Use the initial condition u(x,0)=cos ∏x Homework Equations We eventually get u(x,t)= B0 + ƩBncos(n∏x/L)exp(-n2∏2σ2t/L2) where L=1 and σ=1 in our case. B0 is...
  2. M

    Conditions on overdetermined linear system to be consistent?

    Homework Statement Homework Equations An overtdetermined system - If m > n, then the linear system Ax = b is inconsistent for at least one vector b in ℝ^n. The Attempt at a Solution If m > n (more rows than columns), in which case the column vectors of A cannot span ℝ^m (fewer vectors...
  3. M

    Conditions upon overdetermined linear systems

    Hi All I have an exam tomorrow morning, I've almost completed my study guide but there are a few questions I have no idea how to answer. If someone here could give me a few pointers, or tell me how to solve it, or maybe you already know how to solve it; I can study off your solutions. Any...
  4. R

    FEA Boundary conditions for basic helical spring deformation

    Hello, I was wondering if anyone can help me with my FEA approach. I want to check that my boundary conditions for a simple quarter torus (representing a section of a helical spring) are correct. I'm neglecting the helical angle at this stage. I have fixed one end in all axes, and applied...
  5. B

    Under what conditions does L=Iw hold?

    So, that is the question: Under what conditions does L=Iw hold? Where L,I,w are all scalars. Some specifics perhaps: 1) Supose a fixed axis of rotation, with accel, does is hold? 2) Suppose de axis of rot is not a ppal axis 3) Suppose there is fixed axis of rot, but I choose neither x nor y...
  6. G

    Solving PDEs with Initial & Boundary Conditions

    Homework Statement The PDE: ∂n/∂t + G∂n/∂L=0 The initial condition: n(0,L)=ns The boundary condition: n(t,0)=B/G The parameter B and G above are dependent upon process conditions and change at each time. They can be calculated with adequate experimental data. Homework Equations...
  7. P

    ODE Problem with boundary conditions

    Homework Statement Solve: y'' - λy = 0 where y(0)=y(1)=0, y=y(t) Homework Equations The Attempt at a Solution Hi everyone, This is part of a PDE question, I just need to solve this particular ODE. I know how to do it in the case for y'' + λy = 0, where you get the...
  8. W

    Laplace tranforms with boundary conditions

    Homework Statement Here's the question: Use laplace transforms to find X(t), Y(t) and Z(t) given that: X'+Y'=Y+Z Y'+Z'=X+Z X'+Z'=X+Y subject to the boundary conditions X(0)=2, Y(0)=-3,Z(0)=1. Now I have learned the basics of laplace transforms, but have not seen a question in...
  9. S

    Boundary Conditions for an Infinite Conducting Sheet

    If we place an infinite conducting sheet in free space, and fix its potential to \varphi_0, how do we solve solve for the potential on either side of the sheet? Since the potential blows up at infinity, it seems impossible to define boundary conditions.
  10. D

    Effect of sample size when using periodic boundary conditions in 2D Ising model

    Hi, I'm currently using the Monte Carlo Metropolis algorithm to investigate the 2D Ising model. I have an NxN lattice of points with periodic boundary conditions imposed. I was wondering if anyone could explain why the sharpness of the phase transition is affected by the size of N? I.e...
  11. A

    Equivalent conditions on a metric space

    Homework Statement Let X be a metric space and A a subset of X. Prove that the following are equivalent: i. A is dense in X ii. The only closed set containing A is X iii. The only open set disjoint from A is the empty set Homework Equations N/A The Attempt at a Solution I can...
  12. R

    Basis functions of a differential equation, given boundary conditions

    First off, I've never taken a differential equations class. This is for my Math Methods for Physicists class, and we are on the topic of DE. Unfortunately, we didn't cover this much, so most of what I am about to show you comes from the professor giving me tips and my own common sense. I'd...
  13. S

    Necessary conditions for a linear program

    Homework Statement Consider the following optimization problem: min f(x) s.t. g(x) ≥ 0 h(x) ≤ 0 q(x) = 0  Let xbar satisfy g(x) = h(x) = q(x) = 0. a)State and prove a set of necessary and sufficient conditions for x to be a local minimum. b)How would the conditions...
  14. S

    Double delta potential - boundary conditions

    A double delta potential is given by V(x) = c_+ \delta (x + \frac{L}{2}) + c_- \delta (x - \frac{L}{2}). Use the discontinuity relation to find the boundary conditions in x = \pm \frac{L}{2} . The general solutions are: \psi(x) = \begin{cases} Ae^{ikx} + Be^{-ikx} & x < -\frac{L}{2}...
  15. S

    Heat equation with peculiar boundary conditions

    Homework Statement Find the solution to the heat equation for the following conditions: Homework Equations The Attempt at a Solution Not sure. I've only encountered the following scenarios: temperatures of both ends are arbitrary values both ends are insulated (so the first...
  16. M

    MHB Wave equation and multiple boundary conditions

    I need to apply D'Lembert's method but in this case I don't know how. How to proceed? Determine the solution of the wave equation on a semi-infinite interval $u_{tt}=c^2u_{xx},$ $0<x<\infty,$ $t>0,$ where $u(0,t)=0$ and the initial conditions: $\begin{aligned} & u(x,0)=\left\{ \begin{align}...
  17. T

    Laplace's equation with unusual boundary conditions

    Homework Statement Solve Laplace's equation u_{xx} + u_{yy} = 0 on the semi-infinite domain -∞ < x < ∞, y > 0, subject to the boundary condition that u_y = (1/2)x u on y=0, with u(0,0) = 1. Note that separation of variables will not work, but a suitable transform can be applied...
  18. M

    MHB PDE and more boundary conditions

    Solve $\begin{aligned} & {{u}_{tt}}={{u}_{xx}}+1+x,\text{ }0<x<1,\text{ }t>0 \\ & u(x,0)=\frac{1}{6}{{x}^{3}}-\frac{1}{2}{{x}^{2}}+\frac{1}{3},\text{ }{{u}_{t}}(x,0)=0,\text{ }0<x<1, \\ & {{u}_{x}}(0,t)=0=u(1,t),\text{ }t>0. \end{aligned} $ Here's something new for me, the boundary...
  19. C

    What is the kinematic conditions for free-surface

    When I am studying the Rayleigh-Taylor instability, I saw this equation: \frac{\partial \eta}{\partial t} + u' \frac{\partial \eta}{\partial x} = \omega ' (\eta) I do not quite understand the meaning of this equation. Can some one provide me with some instructions and information...
  20. M

    MHB Solve Heat Equation with Initial Conditions

    Solve $\begin{aligned} & {{u}_{tt}}={{u}_{xx}},\text{ }x\in [0,1],\text{ }t>0, \\ & u(x,0)=f(x), \\ & {{u}_{t}}(x,0)=0,\text{ }u(0,t)=u(1,t)=0 \\ \end{aligned} $ where $f(x)$ is defined by $f(x)=x$ if $0\le x\le \dfrac12$ and $f(x)=1-x$ if $\dfrac12\le x\le1.$ I'm not sure how to...
  21. S

    Boundary conditions - Fresnel equations

    Hello, whenever I come to the derivation of the Fresnel equations I get stuck on the boundary condition for the component of the E-Field that is parallel to the surface. I know for the parallel components Maxwell dictates that: E_{1t} = E_{2t}. For the parallel incoming light field...
  22. M

    Crystal model with periodic boundary conditions

    user meopemuk mentioned this: In the case of a crystal model with periodic boundary conditions, basis translation vectors e1 and e2 are very large (presumably infinite), which means that basis vectors of the reciprocal lattice k1 and k2 are very small, so the distribution of k-points is very...
  23. V

    Why do we need boundary conditions in Physics? Its significance?

    Well as the topics says I need a clarification why do we need the so called boundary conditions? I have seen it in electostatics, magnetostatics etc. I tried in many ways to get that stuff into my head, but its just only banging my head not getting into.. I really wana know what is that and...
  24. B

    Standard conditions vs. Standard state

    What's the difference between standard conditions and standard state? I noticed in my thermodynamics chapter that in standard state, the reaction quotient is 1 because all activities are equal to 1 (if I remember correctly). Standard conditions is about standard temperature and pressure...
  25. A. Neumaier

    Classical solution of PDE with mixed boundary conditions

    Nowadays people usually consider PDEs in weak formulations only, so I have a hard time finding statements about the existence of classical solutions of the Poisson equation with mixed Dirichlet-Neumann boundary conditions. Maybe someone here can help me and point to a book or article where I...
  26. G

    Finding Constants Using Continuity Conditions

    Homework Statement A ball falls from rest at a height H above a lake. Let y = 0 at the surface of the lake. As the ball falls, it experiences a gravitational force -mg. When it enters the water, it experiences a buoyant force B so the net force in the water is B - mg. a) Write an...
  27. N

    Ansys - Boundary conditions for 2 cylinders and fluid

    ansys -- Boundary conditions for 2 cylinders and fluid i want to do a analysis in ansys in which a cylinder will rotate about a axis which is out side of the cylinder and this cylinder is also rotating about its own axis. cylinder is half filled with liduid. i want to do the stress analysis or...
  28. T

    Find the necessary and sufficient conditions on the real numbers a,b,c

    Find the necessary and sufficient conditions on the real numbers a,b,c for the matrix: \begin{bmatrix} 1 & a & b\\ 0 & 1 & c \\ 0 & 0 & 2 \end{bmatrix} to be diagonalizable. Attempt: Now for this one I also solved for the eigenvlues which were: λ1 = 1, λ2 = 1, λ3 = 2 So the problematic...
  29. Q

    Green's functions, F/Laplace Transforms, and Boundary conditions

    I'm having a ton of trouble understanding how to solve diff eqs by using Fourier or laplace transforms to solve for the green's function, with boundary conditions included. I can understand the basics of green's function solutions, especially if transforms are not needed, but my textbook seems...
  30. F

    DE with initial conditions with repeated rootsplease check work thanks

    Homework Statement 4y'' - 4y' + y = 0 y(1) = -4 y'(1) = 0 Homework Equations getting the coeff's gets messier and messier.. am i doing something wrong? The Attempt at a Solution 4y'' - 4y' + y = 0 y'' - y' - (1/4)y = 0 r_1 = r_2 = 1/2 general soln: y(t) = c_1e^(t/2) +...
  31. S

    Monitoring power line conditions

    I would like to do some monitoring of power line conditions. To start with, I just want to monitor and record the voltage wave form on the two split phases. I would dedicate a small computer to the purpose. The idea is to reduce the voltage from the power line in a safe way and feed that...
  32. M

    MHB Transform Boundary Conditions w/ Diff. $g_{0}, g_{1}$

    1) Transform the problem so that boundary conditions turn to homogeneous ones assuming that $g_0$ and $g_1$ are differentiable. $\begin{align} &{{u}_{t}}=K{{u}_{xx}},\text{ }0<x<L,\text{ }t>0, \\ &{{u}_{x}}(0,t)={{g}_{0}}(t),\text{ }{{u}_{x}}(L,t)={{g}_{1}}(t),\text{ for }t>0, \\...
  33. D

    Boundary conditions of solution to the wave equation

    Homework Statement Because q(x,t) = A*exp[-(x-ct)2/σ2] is a function of x-ct, it is a solution to the wave equation (on an infinite domain). (a) What are the initial conditions [a(x) and b(x)] that give rise to this form of q(x,t)? (b) if f(x) is constant, then Eq. (2) shows that solution is...
  34. M

    MHB Another PDE and boundary conditions

    1) Solve $\begin{aligned} {{u}_{t}}&=K{{u}_{xx}},\text{ }0<x<L,\text{ }t>0, \\ {{u}_{x}}(0,t)&=0,\text{ }{{u}_{x}}(L,t)=0,\text{ for }t>0, \\ u(x,0)&=6+\sin \frac{3\pi x}{L} \end{aligned}$ 2) Transform the problem so that the boundary conditions get homogeneous: $\begin{aligned}...
  35. S

    Solving a differential equation with intial conditions,

    Homework Statement Here is the original thing: (x^{2}+1)y'+4x(y-1)=0, y(0)=4 Homework Equations The Attempt at a Solution I thought I knew the procedure.. but I got it wrong. Can someone let me know where I went wrong? First I rearrange the equation to get the following...
  36. K

    Solution of differential with initial conditions

    I think my book is giving me the wrong answer...The problem is to find solution of following: r'(t) = t2\hat{i} + 5t\hat{j} + \hat{k} The initial condition is: r(1) = \hat{j} + 2\hat{k} My solution: r(t) = < (1/3)t3 + c1 , (5/2)t2 + c2 , t+c3 > r(1) = < 0 , 1 , 2 > r(1) = <...
  37. Also sprach Zarathustra

    MHB Show that C.R conditions are held.

    Explain me please how to show that C.R conditions are holds. $$f(z)=\left\{\begin{matrix}e^{-\frac{1}{z^4}}\ \ \ \text{if z not 0 }\\ 0 \ \ \ \ \ \ \ \text{if z not 0 }\end{matrix}\right.$$Thank you.
  38. S

    Really easy PDE, confused about how to put in side conditions

    Hey, before you read this over I'll mention that I've checked the general solution and it works. So if you don't feel like following my steps to get the general solution just jump down to the end of my attempt, because the real problem for me is figuring out what to do with the side conditions...
  39. M

    Solve MATLAB Exercise: Number of Positive Integers < n Divisible by a or b

    Homework Statement Write a Matlab function that takes as input three integers n, a and b, and outputs the number of positive integers less than n that are divisible by a or b. For example, with n = 1000, a = 3 and b = 5, the output should be 466. Homework Equations MATLAB's use of...
  40. C

    Conditions of Stability for second order system

    Homework Statement Consider the transfer function H(s)=\cfrac{1}{a_{2}s^{2}+a_{1}s+a_{0}} where real-valued coefficients a_{2},a_{1}, a_{0} are arbitrary except that a_{2} is nonzero. Verify that the system is stable iff the coefficients a_{2},a_{1}, a_{0} have the same sign. Homework...
  41. S

    Magnetism: conditions for magnetic attraction and magnetic shielding

    Hi, I have two questions that come from a unique example of magnetism. Here is the scenario: a thin and long piece of iron is placed in between a magnet and the north pole faces the outing sheet and some iron tacks. The iron tack do not get attracted to the iron even though a south pole is...
  42. B

    Cauchy-Riemann Conditions in Polar Coordinates

    Homework Statement Using f(z) = f(re^iθ) = R(r,θ)e^iΩ(r,θ), show that the Cauchy-Riemann conditions in polar coordinates become ∂R/∂r = (R/r)∂Ω/∂θ Homework Equations Cauchy-Riemann in polar coordinates Hint: Set up the derivative first with dz radial and then with dz tangential...
  43. F

    Normalization Conditions of Wave functions

    I am currently reading through Griffiths Quantum Mechanics textbook, and on page 14, Griffiths proves that \frac{d}{dt}\int_{-\infty}^{\infty} |\Psi(x,t)|^2 \, dx = \left.\frac{i \hbar}{2m}\left( \Psi^* \frac{\partial \Psi}{\partial x} - \frac{\partial \Psi^*}{\partial x} \Psi \right)...
  44. R

    Solving Non-Linear Diffusion Eqn w/ Eigenfunction Expansion

    I'm trying to solve a non-linear time-dependent diffusion equation to find R(x,t). To do so, I'm positing that : R(x,t)=\sum^{J}_{1}X_{i}(x)T_{i}(t) which allows me to arrive at something that looks like : dT_{i}/dt=A_{i}T_{i}(t)-B*T_{i}(t)^{2} The problem I'm having, through sheer...
  45. M

    Solving Homogenous Eq. with Initial Conditions: u(x)=\int^x_0 f(s)g(x,s)ds

    Eq u'(x)+p(x)u=f(x) with initial condition u(0)=0 It's homogenous solution is u_h=Ce^{-\int^x_0 p(s)ds} Complete solution u(x)=e^{-\int^x_0 p(s)ds}\int^x_0f(s)e^{\int^s_0 p(z)dz}ds=\int^x_0 f(s)g(x,s)ds where g(x,s)=e^{-\int^{x}_s p(\xi)d \xi } I didn't see that last...
  46. fluidistic

    Neumann boundary conditions on a PDE

    Homework Statement From a previous exercise (https://www.physicsforums.com/showthread.php?t=564520), I obtained u(r,\phi) = \frac{1}{2}A_{0} + \sum_{k = 1}^{\infty} r^{k}(A_{k}cos(k\phi) + B_{k}sin(k\phi)) which is the general form of the solution to Laplace equation in a disk of radius a. I...
  47. K

    (Image Theory) Boundary conditions and how many image charges do i need?

    Homework Statement An electrostatic point charge of 1 Coulomb (C) placed symmetrically between two infinitely/perfectly conducting parallel plates. These two infinitely large conducting plates are parallel to the yz plane.The region between the two plates is designated as “Region A.” Starting...
  48. maverick_starstrider

    Classical Stat Mech with Uncertain Initial Conditions vs. Quantum

    Hi, I was wondering if someone could point me to a textbook or easy to read paper (or website) that briefly describes/proves the differences here. What I mean is if I do classical (continuous energies) statistical mechanics where my initial state is a volume (greater than or equal to h-bar)...
  49. D

    Conditions of igneous rocks on the early Earth

    I have a question that's related to radiometric dating. I would like to know what the condition would be of igneous rocks in their early states. What I mean is the state in which the radioactive elements found in the rocks, which are used for dating, have not yet undergone any decay. For...
  50. V

    Neumann Boundary Conditions question

    So I'm reading through Jackson's Electrodynamics book (page 39, 3rd edition), and they're covering the part about Green's theorem, where you have both \Phi and \frac{\delta \Phi}{\delta n} in the surface integral, so we often use either Dirichlet or Neumann BC's to eliminate one of them. So for...
Back
Top